Unitary Operators and Lorentz Transformations

In summary, Srednicki's QFT book is difficult for me because I don't understand what the unitary operator does--what the details are. I am stuck in chapter 2 and 3 and need help to figure out how to solve the equations. I think that the right hand side of the equation might give me the result I'm looking for.
  • #1
abode_x
11
0

Homework Statement


I am trying to learn from Srednicki's QFT book. I am in chapter 2 stuck in problem 2 and 3. This is mainly because I don't know what the unitary operator does - what the details are.

Starting from:
[tex]U(\Lambda)^{-1}U(\Lambda')U(\Lambda)=U(\Lambda^{-1}\Lambda'\Lambda)[/tex]
How does one arrive at:
[tex]\delta\omega_{\mu\nu}U(\Lambda)^{-1}M^{\mu\nu}U(\Lambda)=\delta\omega_{\mu\nu}\Lambda^{\mu}_{\rho}\Lambda^{\nu}_{\sigma}M^{\mu\nu}[/tex]

Homework Equations


Given that:
[tex]\Lambda '=1+\delta\omega [/tex]

[tex]U(1+\delta\omega)=I + \frac{i}{2 \hbar}\delta\omega_{\mu\nu}M^{\mu\nu}[/tex]

The Attempt at a Solution


Working out the left hand side from the given, I end up with
[tex]I+\frac{i}{2 \hbar}\delta\omega_{\mu\nu}U(\Lambda)^{-1}M^{\mu\nu}U(\Lambda)[/tex]

As for the RHS, I don't know the details. Why do I end up with those contractions of two Lorentz transformations with the generator of the Lorentz group M?

Also, since I'm already here, I would also like to ask what is meant by,
[tex][M^{\mu\nu},M^{\rho\sigma}]=i\hbar (g^{\mu\rho}M^{\mu\nu} - (\mu \leftrightarrow \nu) ) - ( \rho \leftrightarrow \sigma ) [/tex]

specifically the notation with the double arrow. It seems like an index replacement?
 
Last edited:
Physics news on Phys.org
  • #2
Yes, it's a shorthand notation for an index replacement. And correct the line in which you wrote what you're trying to prove. See how the summations are done. I'll be doing the calculations for myself an see if i get to the result you're supposed to prove.
 
Last edited:
  • #3
Here's what is get

[tex]U\left( \Lambda \right) ^{-1}\delta \omega _{\mu \nu }M^{\mu \nu }U\left( \Lambda \right) =\left( \Lambda ^{-1}\right) _{\rho }{}^{\mu }\delta \omega _{\mu \nu }\Lambda ^{\nu }{}_{\sigma }M^{\rho \sigma } [/tex]

Now where does the result you quote follow from ?? The result you're quoting appears also in the books by Weinberg and Gross. But i don't see how my expression translates to what they all are saying, namely

[tex] \delta \omega _{\mu \nu }U\left( \Lambda \right) ^{-1}M^{\mu \nu }U\left( \Lambda \right) =\delta \omega _{\mu \nu }\Lambda^{\mu }{}_{\rho }\Lambda ^{\nu }{}_{\sigma }M^{\rho \sigma }[/tex]

Hmmm.
 
  • #4
Starting from:
[tex]U(\Lambda)^{-1}U(\Lambda')U(\Lambda)=U(\Lambda^{-1}\Lambda'\Lambda)[/tex]

It says to plug in [tex] \Lambda '=1+\delta\omega[/tex], to first order in [tex]\delta\omega[/tex]

Working out the left hand side from the given, I end up with
[tex]I+\frac{i}{2 \hbar}\delta\omega_{\mu\nu}U(\Lambda)^{-1}M^{\mu\nu}U(\Lambda)[/tex]

My guess is that the right hand side would give me
[tex]I+\frac{i}{2 \hbar}\delta\omega_{\mu\nu}\Lambda^{\mu}_{\rho}\Lambda^{\nu}_{\sigma}M^{\rho\sigma}[/tex]

So that i get the result. I have no idea how to get the contractions. My thinking is that [tex]\Lambda^{-1}\Lambda'\Lambda[/tex] is a matrix representation for a change of basis that is why the M comes out in [tex]\rho[/tex] and [tex]\sigma[/tex]. But why does delta stay in [tex]\mu[/tex] and [tex]\nu[/tex]?

The result we want is:
[tex]\delta\omega_{\mu\nu}U(\Lambda)^{-1}M^{\mu\nu}U(\Lambda)
=\delta\omega_{\mu\nu}\Lambda^{\mu}_{\rho}\Lambda^{\nu}_{\sigma}\underline{M^{\rho\sigma}}[/tex]

Note the typo I made in my first post (underlined).
 
Last edited:
  • #5
dextercioby said:
Here's what is get

[tex]U\left( \Lambda \right) ^{-1}\delta \omega _{\mu \nu }M^{\mu \nu }U\left( \Lambda \right) =\left( \Lambda ^{-1}\right) _{\rho }{}^{\mu }\delta \omega _{\mu \nu }\Lambda ^{\nu }{}_{\sigma }M^{\rho \sigma } [/tex]

Now where does the result you quote follow from ?? The result you're quoting appears also in the books by Weinberg and Gross. But i don't see how my expression translates to what they all are saying, namely

[tex] \delta \omega _{\mu \nu }U\left( \Lambda \right) ^{-1}M^{\mu \nu }U\left( \Lambda \right) =\delta \omega _{\mu \nu }\Lambda^{\mu }{}_{\rho }\Lambda ^{\nu }{}_{\sigma }M^{\rho \sigma }[/tex]

Hmmm.

According to Srednicki's eq.(2.5), [tex](\Lambda ^{-1}) _\rho{}^\mu = \Lambda^\mu{}_\rho[/tex], so you're done.
 
  • #6
Yes, you're right. The inverse of a Lorentz matrix is the transposed matrix and then the indices can be shuffled.
 

1. What is a unitary operator?

A unitary operator is a mathematical concept used in quantum mechanics to describe transformations that preserve the inner product of a vector space. In simpler terms, it is an operator that does not change the length or direction of a vector.

2. How are unitary operators used in quantum mechanics?

Unitary operators are used to describe the evolution of quantum systems over time. They are used to transform the state of a quantum system from one point in time to another, and they play a crucial role in calculating the probabilities of different outcomes in quantum experiments.

3. What is the significance of unitary operators being reversible?

The fact that unitary operators are reversible means that the transformation can be undone, allowing for the original state of the system to be retrieved. This is important in quantum computing, where the reversibility of operations is necessary to ensure the accuracy of calculations.

4. What are Lorentz transformations?

Lorentz transformations are a set of equations used in special relativity to describe how measurements of space and time change for an observer moving at a constant velocity relative to another observer. They involve the concepts of time dilation, length contraction, and the relativity of simultaneity.

5. How are unitary operators related to Lorentz transformations?

Unitary operators and Lorentz transformations are both mathematical concepts used in different areas of physics. However, in quantum field theory, unitary operators are used to describe the symmetries of a system, while Lorentz transformations are used to describe the symmetries of space and time in special relativity.

Similar threads

  • Advanced Physics Homework Help
Replies
2
Views
441
  • Advanced Physics Homework Help
Replies
22
Views
3K
  • Advanced Physics Homework Help
Replies
1
Views
298
  • Advanced Physics Homework Help
Replies
1
Views
1K
  • Advanced Physics Homework Help
Replies
3
Views
1K
  • Advanced Physics Homework Help
Replies
1
Views
2K
  • Advanced Physics Homework Help
Replies
2
Views
2K
  • Advanced Physics Homework Help
Replies
8
Views
712
Replies
6
Views
2K
  • Advanced Physics Homework Help
Replies
2
Views
959
Back
Top